Από Τουρκία

Συντονιστές: cretanman, silouan, rek2

2nisic
Δημοσιεύσεις: 220
Εγγραφή: Παρ Δεκ 04, 2020 12:06 pm

Από Τουρκία

#1

Μη αναγνωσμένη δημοσίευση από 2nisic » Σάβ Ιούλ 24, 2021 2:02 pm

Να βρεθούν όλοι οι φυσικοί n,m,k με k\geq 2 τέτοιοι ώστε:
2^{2n+1}+2^{n}+1=m^k


Σχόλιο:Το περίεργο είναι ότι η άσκηση τέθηκε στην Τουρκία το 2003 και στην imo σε ποίο εύκολη μορφή (k=even) το 2006



Λέξεις Κλειδιά:
Άβαταρ μέλους
Demetres
Γενικός Συντονιστής
Δημοσιεύσεις: 8989
Εγγραφή: Δευ Ιαν 19, 2009 5:16 pm
Τοποθεσία: Λεμεσός/Πύλα
Επικοινωνία:

Re: Από Τουρκία

#2

Μη αναγνωσμένη δημοσίευση από Demetres » Πέμ Ιούλ 29, 2021 12:13 pm

Για n=0 έχω τη λύση n=0,m=2,k=2. Για n=1 είναι άμεσο ότι δεν υπάρχει λύση.

Για n \geqslant 2 έχω

\displaystyle  2^{2n+1} + 2^n = m^k-1 = (m-1)(m^{k-1} + \cdots + m + 1)

Αν k περιττός, τότε m^{k-1} + \cdots + m + 1 επίσης περιττός. Άρα 2^n | m-1. Τότε m \geqslant 2^n οπότε m^k \geqslant m^3 \geqslant 2^{3n} \geqslant 2^{2n+2} > 2^{2n+1} + 2^n + 1.

Αυτό είναι άτοπο επομένως ο k είναι άρτιος, έστω k = 2\ell. Τότε

\displaystyle  m^{2\ell} = 2^{2n+1} + 2^n + 1 = (2^{n-1}+1)^2 + 7 \cdot 2^{2n-2}

Άρα

\displaystyle  (m^{\ell} - 2^{n-1}-1)(m^{\ell} + 2^{n-1}+1) = 7 \cdot 2^{2n-2}

Έχουμε

\displaystyle  (m^{\ell} - 2^{n-1}-1) + (m^{\ell} + 2^{n-1}+1) = 2m^{\ell} \equiv 2 \bmod 4

Επειδή το γινόμενο των δύο αριθμών είναι άρτιο και το άθροισμα είναι ισότιμο με 2 \bmod 4 πρέπει και οι δύο να είναι άρτιοι με τουλάχιστον τον ένα από τους δύο να μην είναι πολλαπλάσιος του 4. Πρέπει λοιπόν να έχουμε μια από τις πιο κάτω περιπτώσεις:
  • m^{\ell} - 2^{n-1}-1 = 2 και m^{\ell} + 2^{n-1}+1 = 7 \cdot 2^{2n-3}
  • m^{\ell} - 2^{n-1}-1 = 14 και m^{\ell} + 2^{n-1}+1 = 2^{2n-3}
  • m^{\ell} - 2^{n-1}-1 = 2^{2n-3} και m^{\ell} + 2^{n-1}+1 =14
Στην πρώτη περίπτωση έχουμε 2(2^{n-1}+1) = 7 \cdot 2^{2n-3} -2 \geqslant 7 \cdot 2^{n-1}-2 που είναι άτοπο αφού δίνει 4 \geqslant 5 \cdot 2^{n-1} \geqslant 5.

Στη δεύτερη περίπτωση έχουμε 2(2^{n-1}+1) = 2^{2n-3} - 14. Για n \leqslant 3 το δεξί μέλος είναι μικρότερο του 0, άτοπο. Για n=4 παίρνουμε m^k = 529 που δίνει τη λύση n=4,m=23,k=2. Για n \geqslant 5 έχουμε 2(2^{n-1}+1) = 2^{2n-3} - 14 > 8 \cdot 2^{n-1} - 14 που δίνει 16 \geqslant 6 \cdot 2^{n-1} \geqslant 96, άτοπο.

Στην τρίτη περίπτωση έχουμε 2(2^{n-1}+1) = 14 - 2^{2n-3} . Για να είναι το δεξί μέλος θετικό πρέπει n=2 ή n=3 που εύκολα απορρίπτονται.

Άρα μοναδικές λύσεις οι (n,m,k) = (0,2,2) και (n,m,k) = (4,23,2).


Απάντηση

Επιστροφή σε “Θεωρία Αριθμών - Επίπεδο Αρχιμήδη (Seniors)”

Μέλη σε σύνδεση

Μέλη σε αυτήν τη Δ. Συζήτηση: Δεν υπάρχουν εγγεγραμμένα μέλη και 2 επισκέπτες